Question

2.100 = 1) -- 1 -- 1) --- ) . w in the subopac spumed by you [1] 2. Let v1 = 0 V2 = -1 not? Justify your answer. Is w in the
0 0
Add a comment Improve this question Transcribed image text
Request Professional Answer

Request Answer!

We need at least 10 more requests to produce the answer.

0 / 10 have requested this problem solution

The more requests, the faster the answer.

Request! (Login Required)


All students who have requested the answer will be notified once they are available.
Know the answer?
Add Answer to:
2.100 = 1) -- 1 -- 1) --- ) . w in the subopac spumed by...
Your Answer:

Post as a guest

Your Name:

What's your source?

Earn Coins

Coins can be redeemed for fabulous gifts.

Similar Homework Help Questions
  • 1) Determine if w is in the subspace spanned by v1, v2, v3 2) Are the...

    1) Determine if w is in the subspace spanned by v1, v2, v3 2) Are the vectors v1, v2, v3 linearly dependent or independent? justify your answer Question 2. (15 pts) Let vi=(-3 0 6)", v2= (-2 2 3]", V3= (0 - 6 37, and w= [1 11 9". (1). Determine if w is in the subspace spanned by V1, V2, V3. (2). Are the vectors V1, V2, V3 linearly dependent or independent? Justify your answer

  • Let v1= [−3 0 6]T , v2= [−2 2 3]T , v3= [0 − 6 3]T...

    Let v1= [−3 0 6]T , v2= [−2 2 3]T , v3= [0 − 6 3]T , and w= [1 14 9]T . (1). Determine if w is in the subspace spanned by v1, v2, v3. (2). Are the vectors v1, v2, v3 linearly dependent or independent? Justify your answer.

  • Question 2. (15 pts) Let Vi= (-3 0 6)", v2= (-2 2 3)", V3= [0 -...

    Question 2. (15 pts) Let Vi= (-3 0 6)", v2= (-2 2 3)", V3= [0 - 6 3)", and w= [1 14 9)? (1). Determine if w is in the subspace spanned by V1, V2, V3. (2). Are the vectors Vi, V2, V3 linearly dependent or independent? Justify your answer.

  • Question 2. (15 pts) Let vi= (-3 0 6)", V2= (-2 2 317, V3= [0 -...

    Question 2. (15 pts) Let vi= (-3 0 6)", V2= (-2 2 317, V3= [0 - 6 3)", and w=(1 14 9) (1). Determine if w is in the subspace spanned by va, V2, V3. (2). Are the vectors V1, V2, V3 linearly dependent or independent? Justify your answer.

  • Question 2. (15 pts) Let vi= [-3 0 6)". Vy=[-2 2 3". Vg= [0 - 6...

    Question 2. (15 pts) Let vi= [-3 0 6)". Vy=[-2 2 3". Vg= [0 - 6 3), and w=[1 14 97 (1). Determine if w is in the subspace spanned by V. V2 V3. (2). Are the vectors Vi, V2, V3 linearly dependent or independent? Justify your answer.

  • Homework: Section 4.1 Score: 0 of 1 pt 4.1.13 3 4 11 Let v1 0 ,...

    Homework: Section 4.1 Score: 0 of 1 pt 4.1.13 3 4 11 Let v1 0 , V2|1 V3 3 and w= 1 1 4 10 Is w in {v,, v2, v3}? How many vectors are in {v,, v, V3}? b. How many vectors are in Span{v, V2, V3}? c. Is w in the subspace spanned by (v,, v2, V3)? Why? a a. Is w in {v, V2 V31? O A. Vector w is in {v,, v2, V3} because it is...

  • I am looking for how to explain #4 part b. I have gotten the matrix A...

    I am looking for how to explain #4 part b. I have gotten the matrix A and I believe the answer is W = span{ v1 u2 u3 } however I'm not really sure if that is correct or not. Please give a small explanation. Also im not sure if I need to represent the vectors in A as columns or rows, or if either one works. For the next two problems, W is the subspace of R4 given by...

  • Problem #18: [2 marks] Let W be the subspace of R4 spanned by the vectors u...

    Problem #18: [2 marks] Let W be the subspace of R4 spanned by the vectors u - (1,0,1,0), u2 = (0.-1, 1.0), and ug = (0.0, 1,-1). Use the Gram-Schmidt process to transform the basis (uj, u, uz) into an orthonormal basi (A) v1 = (-12,0, 2.0), v2 - (VG VG VG, o), v3 - (I ) (B) v1 = (-V2.0, .), v2 - (VG VG VG o), v3 - (™J - V3 VI-V3) (C) v1 - ($2.0, 92.0), v2...

  • (a) (5 points.) Let W CW CW CW3 be distinct subspaces of R? True/False (Justify your...

    (a) (5 points.) Let W CW CW CW3 be distinct subspaces of R? True/False (Justify your answers): (i) Wo must be the zero subspace. (ii) W, must be R. (iii) W, must be RP. (iv) Suppose V1, V2, V3 are vectors such that vi EWW -for each 1 <i<3. Then {V1, V2, V3} must be a basis for R. (v) There are three linearly independent vectors in R that do not form a basis for R?

  • -9 2. Let Vi-8.V2,andvs-2, let B -(V,V2,Vs), and let W be the subspace spanned , let B -(Vi,V2,V3...

    -9 2. Let Vi-8.V2,andvs-2, let B -(V,V2,Vs), and let W be the subspace spanned , let B -(Vi,V2,V3), and let W be the subspace spanned by B. Note that B is an orthogonal set. 17 a. 1 point] Find the coordinates of uwith respect to B, without inverting any matrices or L-2 solving any systems of linear equations. 35 16 25 b. 1 point Find the orthogonal projection of to W, without inverting any matrices or solving any systems of...

ADVERTISEMENT
Free Homework Help App
Download From Google Play
Scan Your Homework
to Get Instant Free Answers
Need Online Homework Help?
Ask a Question
Get Answers For Free
Most questions answered within 3 hours.
ADVERTISEMENT
ADVERTISEMENT
ADVERTISEMENT